#26 Rosh Review

Pataasin ang iyong marka sa homework at exams ngayon gamit ang Quizwiz!

A previously healthy 71-year-old woman presents to your office with complaints of white spots in her mouth and a sore tongue. Physical exam reveals dentures, white plaques on the buccal mucosa and tongue, and erythematous lesions on the hard palate. Which of the following is the most appropriate therapy? Oral acyclovir Oral ibuprofen Oral nystatin suspension Oral viscous lidocaine solution

Correct Answer ( C ) Explanation: Oropharyngeal candidiasis, also called thrush, is a local infection commonly seen in older adults who wear dentures, infants, individuals with diabetes, immunocompromised patients, and as a side effect of antibiotics, chemotherapy, or inhaled glucocorticoids. There are two forms of oropharyngeal candidiasis. Clinical manifestations with the pseudomembranous form include white plaques on the palate, tongue, buccal mucosa or oropharynx. The atrophic form presents as erythema without plaques and is seen frequently in patients who wear dentures. Oropharyngeal candidiasis may also present as a beefy red tongue and patients may complain of soreness. Diagnosis is with potassium hydroxide preparation. First-line treatment is with topical antifungals, such as nystatin oral suspension. Patients with recurrent symptoms should be tested for the human immunodeficiency virus. Patients with dentures should be counseled to clean their dentures thoroughly and frequently to prevent future infections. Oral viscous lidocaine solution (D) is an analgesic used for symptomatic treatment of oral pain. It is often used for patients with oral lesions associated with chemotherapy or medical procedures. Oral acyclovir (A) is an antiviral used in the treatment of herpes simplex, which can present with intraoral vesicular lesions on an erythematous base. Oral ibuprofen (B) may be used as supportive treatment to help with pain, but does not treat the cause of the lesions.

A runner presents with recurrent anterior knee pain. She reports a "popping" sensation followed by severe pain. These episodes are brief because the pain resolves once she moves her "kneecap back into position". In the initial evaluation of this patient, a radiologic order would most likely include which of the following views? Frogleg view Grashey view Sunrise view Swimmer's view

Correct Answer ( C ) Explanation: Patellofemoral instability is the transient displacement (usually laterally) of the patella, either partially (subluxation) or completely (dislocation), causing acute or chronic patellar pain. It is associated with a positive apprehension sign when displacing the patella laterally. Diagnosis necessitates AP, lateral, tunnel and axial views. The axial view, also commonly called the sunrise view, allows the clinician a direct visualization of how the patella sits in the femoral trochlear groove. Normally, the "train is on the tracks", in other words, the patella sits equidistantly between the femoral condyles. In patients with malalignment or instability, the "train is off the tracks", showing the patella riding, most commonly, over the lateral femoral condyle, outside of the trochlear groove. Acute cases are treated with rest, icing, splinting and compression, but may also require manual reduction. Chronic recurrent cases require aggressive physical therapy and intermittent bracing. The frogleg lateral view (A) is used in evaluating the hip, not the knee. The Grashey view (B) is used in evaluating the glenohumeral joint, not the knee. A swimmer's view (D) is used to evaluate the cervicothoracic junction, not the knee.

Which of the following confirms an intrauterine pregnancy? Beta-hCG of 200,000 Fetal heart activity Intrauterine fetal pole and yolk sac Single layer intrauterine gestational sac

Correct Answer ( C ) Explanation: The fetal pole is a mass of fetal cells separate from the yolk sac that first becomes apparent on transvaginal ultrasound just after the 6th week of gestation. It is the fetus in its somite stage. Usually, you can identify rhythmic fetal cardiac movement within the fetal pole, although it may need to grow several millimeters before this is apparent. Identification of an intrauterine fetal pole and yolk sac on ultrasound confirms an intrauterine pregnancy. Other ways that increase the likelihood, but does not confirm an intrauterine pregnancy include identifying the double ring, also known as the double decidual sign, which usually appears by 5.5-6 weeks' gestation. The double ring helps to distinguish a true gestational sac from an intrauterine fluid collection or pseudosac. Identifying intrauterine fetal heart activity confirms an intrauterine pregnancy. Beta-hCG levels (A) increase at a predictable rate in the early stages of pregnancy. Levels of beta-hCG double every 2-3 days during the first 7-8 weeks of normal pregnancies. However, there are conditions other than an intrauterine pregnancy that can lead to significant elevations of beta-hCG, such as gestational trophoblastic disease and molar pregnancy. This disorder is characterized by proliferation of chorionic villi. The associated high level of beta-hCG often leads to hyperemesis. Identifying fetal heart activity (B) does not confirm an intrauterine pregnancy because an ectopic pregnancy can progress to have fetal activity. Only when fetal heart activity is detected within the gestational sac can the pregnancy be confirmed as intrauterine. A true gestational sac is identified on ultrasound by the double ring, also known as the double decidual sign, and confirms an intrauterine pregnancy. A single layer gestational sac (D), or pseudosac, is possible, which looks very similar to a true gestational sac. In this setting, there can still be an ectopic fetus. Therefore, most clinicians confirm an intrauterine pregnancy when a gestational sac with a fetal pole or yolk sac is present.

A 19-year-old man presents to the ED after a syncopal event. The patient states that he was moving a large pile of wood in his backyard and then suddenly became dizzy. He awoke with his neighbor standing over him asking if he was okay. On exam, you note a loud crescendo-decrescendo systolic murmur at the left lower sternal border. He has a family history of a "heart condition." Which of the following maneuvers will accentuate the murmur? Leg elevation Squatting Standing Trendelenburg

Correct Answer ( C ) Explanation: This patient has hypertrophic cardiomyopathy, which is characterized by asymmetric thickening of the left ventricular septal wall. On exam, it is associated with a loud S4 gallop and a harsh crescendo-decrescendo midsystolic murmur. Unlike the harsh murmur of aortic stenosis, which may sound similar, the murmur associated with hypertrophic cardiomyopathy does not typically radiate to the neck. Various maneuvers can help differentiate the murmurs as well. The murmur of hypertrophic cardiomyopathy is accentuated by actions that decrease blood volume in the left ventricle and increase left ventricular outflow obstruction. These actions include the Valsalva maneuver or standing. Leg elevation (A), squatting (B), and Trendelenburg (D) are all associated with an increase in left ventricular blood volume and a consequent decrease in both subvalvular outflow obstruction and intensity of the associated murmur.

A 44-year-old man with end stage renal disease presents from dialysis complaining of weakness and shortness of breath. He states that he missed his last dialysis session, and today they thought he was too sick and sent him to the ED. His vitals are T 37.5°C, HR 73, and BP 89/52. A 12-lead ECG is performed, as seen above. Which of the following represents the appropriate management of this patient? Aggressive intravenous fluids, broad-spectrum antibiotics, and admission to the ICU Intravenous amiodarone Intravenous calcium chloride Sedation and electrical cardioversion

Correct Answer ( C ) Explanation: This patient is suffering from severe hyperkalemia. The ECG shows a regular rhythm with a prolonged PR interval, wide QRS complex, and peaked T waves. In addition to the ECG, the patient's history of missed dialysis sessions supports the diagnosis. The immediate treatment of unstable patients with hyperkalemia should be intravenous calcium as either calcium gluconate or calcium chloride. One gram of calcium gluconate contains 1/3 of the amount of calcium as 1 gram of calcium chloride and thus, if calcium gluconate is used, 3 grams should be administered. Calcium acts by stabilizing cardiac myocytes and acts within minutes. Electrical cardioversion (D) and intravenous amiodarone (B) are appropriate for the treatment of ventricular tachycardia (another wide-complex rhythm). This patient has a wide-complex rhythm, but it is too slow to be ventricular tachycardia. In addition, P waves are present, which is not seen in ventricular tachycardia. Amiodarone is reserved for ventricular tachycardia in patients that are stable. Aggressive intravenous fluids and antibiotics (A) would be the treatment for sepsis or septic shock, a common source of hypotension in patients with end stage renal disease.

A 25-year-old man with a history of hereditary angioedema presents with swelling of the tongue and lips for 30 minutes. The patient appears to have difficulty breathing if he lies flat but is comfortable when sitting up. Visualization of the mouth and posterior pharynx is completely obstructed by the tongue. What management should be initiated? Diphenhydramine Epinephrine 1:1000 intramuscular Fresh frozen plasma infusion Methylprednisolone

Correct Answer ( C ) Explanation: This patient presents with hereditary angioedema. Angioedema reactions, in general, are similar pathophysiologically to urticaria but involve deeper dermal and subcutaneous tissues. However, when angioedema presents in the absence of hives, it suggests involvement of bradykinin as opposed to mast cell degranulation. Hereditary angioedema (HAE) is due to a deficiency of C1 esterase inhibitor. In cases of severe angioedema, fresh frozen plasma (FFP) has been used with successful results. FFP replaces C1 esterase inhibitor and contains kininase II, which breaks down bradykinin. Epinephrine (B), diphenhydramine (A) and methylprednisolone (D) are effective in the treatment of allergic reactions and anaphylaxis but not in the treatment of angioedema. Epinephrine's alpha-agonist effects increase peripheral vascular resistance, decrease vascular permeability and reduce systemic hypotension. Beta-agonist effects lead to bronchodilation and increased cardiac inotropy and chronotropy. Diphenhydramine is a histamine 1 receptor blocker and blocks the peripheral effects of histamine. Steroids, like methylprednisolone, stabilize mast cells leading to decreased release of inflammatory mediators involved in allergic reactions.

A 32-year-old woman presents to your office with a complaint of productive cough that started 6 days ago. She reports occasional wheezing, chest wall tenderness, and has been afebrile since the onset of symptoms. Her husband had similar symptoms and was prescribed azithromycin by his primary care provider. Which of the following is the most appropriate next step in management? Order a chest X-ray Prescribe a course of azithromycin Send a sputum sample for culture Symptomatic treatment

Correct Answer ( D ) Explanation: Acute bronchitis is an inflammation of the bronchi caused by upper respiratory infection. It is a self-limiting condition that is most commonly of viral etiology. Patients with acute bronchitis generally have few systemic symptoms. The most common clinical presentation is a cough lasting up to 10-20 days. The cough may be productive and patients often mistakenly attribute purulent sputum in acute bronchitis with a bacterial infection. The symptoms of acute bronchitis are less severe than those of influenza or pneumonia and help determine diagnosis and course of treatment. Symptomatic treatment with acetaminophen or NSAIDs and other supportive care measures are recommended as initial management of patients with acute bronchitis. Chest X-ray (A) is used to rule out pneumonia and should be ordered when a patient has abnormal vital signs including temperature greater than 38°C, pulse greater than 100/min, respirations greater than 24/min, or age greater than 75 years with more severe clinical symptoms. Antibiotics (B) are usually not indicated due to the viral nature of acute bronchitis, but are often prescribed. Acute bronchitis is one of the most common reasons for antibiotic abuse. Sending a sputum sample for culture (C) is not recommended for patients with acute bronchitis. Culture may be indicated in cases where pertussis is suspected.

A 19-year-old man presents to the emergency department with acute dysuria 3 days after a weekend of heavy drinking. He admits to sexual intercourse with 3 different women during this weekend of partying. Examination reveals yellow discharge from the urinary meatus. Which of the following is the most likely diagnosis? Balanitis Cystitis Pyelonephritis Urethritis

Correct Answer ( D ) Explanation: Males exposed to sexually transmitted infections commonly present emergently with urethritis. Although 16% of those with bacterial infection of the urethra are asymptomatic, most patients present with urethral discharge, dysuria, painful intercourse and painful ejaculation. Other common findings are hematuria, urethral pruritus and hemospermia. Urethritis is characterized by urethral discharge, penile itching or tingling, dysuria, urgency and hematuria. Exam is notable for urethral discharge, penile ulcers, and inguinal lymphadenopathy. Neisseria gonorrhoeae is the cause of gonoccocal urethritis. Causes of nongonoccocal urethritis include Mycoplasma genitalium, Ureaplasma, Trichomonas, HSV, and adenovirus. Confirmatory testing is necessary as gonorrhea and chlamydia infections must be reported to state health departments. Options include gram stain, urine culture, direct immunofluorescence, immunoassay, polymerase chain reaction, nucleic acid hybridization or nucleic acid amplification testing. In the emergency setting where test results are typically unavailable, presumptive treatment of both gonococcal urethritis with IM ceftriaxone and nongonococcal urethritis with 7 days of oral doxycycline is common practice. Cystitis (B) or prostatitis should be considered higher in the differential if dysuria and urethral discharge is accompanied by urinary frequency or urgency. Pyelonephritis (C) is uncommon in young men. However, it is characterized by dysuria, fever, and flank pain. Balanitis (A) is inflammation of the glans penis, usually due to fungal organisms. It presents with burning, irritation, and redness of the head of the penis. Diabetes is the most common condition underlying balanitis. It is not associated with urethral discharge.

A 79-year-old woman with Parkinson's disease is being treated with three-times-a-day carbidopa/levodopa for the past 5 years. She recently developed cervical dystonia and involuntary tongue movements, both of which have caused feeding impairments. She does not report problems with tremor or rigidity. Her medical history is significant for constipation and hypotensive episodes. Which of the following pharmacologic interventions is recommended at this time? Add benztropine, and switch to five-times-a-day dosing of carbidopa/levodopa Add nothing, but switch any immediate-release carbidopa/levodopa to a controlled-release formulation Add ropinirole, and switch to once-a-day dosing of carbidopa/levodopa Continue the current carbidopa/levodopa dosing, but add pramipexole

Correct Answer ( D ) Explanation: Parkinson's disease is mainly characterized by its impairment on voluntary movement, leading to gait, balance and fine motor abnormalities which significantly affects functionality. Carbidopa/levodopa (immediate-release) is the most effective medication for this disease, and is the mainstay first-line medication used, especially early in the disease course (<5 years). Unfortunately, long-term use of carbidopa/levodopa can cause tardive dyskinesia and further motor control complications, which may cause further functional limitations, in up to 40% of patients treated with this drug for >5 years. As disease progresses, increasing the dose of carbidopa/levodopa is common practice. Dopamine agonists, such as bromocriptine, pramipexole and ropinirole, are also considered first-line treatment of motor abnormalities, albeit they have less benefit than carbidopa/levodopa, but fortunately have less motor side effects than levodopa. Overall, when carbidopa/levodopa has reached its end usefulness, there are several options to consider. Some clinicians will split up the total daily carbidopa/levodopa dose into 5 smaller doses a day. Others will decrease the carbidopa/levodopa dose, and some will convert any controlled-release preparations into immediate-release preparations. Another viable option is to add, or switch to adjuvant medications like dopamine-agonists (pramipexole). Other adjuvants include monoamine oxidase-B inhibitors like selegiline, catechol-O-methyltransferase inhibitors like tolcapone and entacapone, and anticholinergics like benztropine. The most sensible option above is to add an adjuvant medication. Although increasing the carbidopa/levodopa dosing frequency is recommended, anticholinergics (A) are likely not a viable choice in this scenario, as their common side effects of dry mouth, dry eyes, constipation, hypotension, urinary retention and cognitive impairment, especially in the elder population, limit their use. When fraught with carbidopa/levodopa side effects, convert all controlled-release forms into immediate-release forms (B), not the opposite. Although adding a dopamine-agonist like ropinirole (C) is recommended, decreasing the carbidopa/levodopa dosing frequency is not.

A 72-year-old man is sent to the interventional radiology department by his primary care doctor to undergo a thoracentesis after a chest radiograph revealed a moderate left-sided pleural effusion. Analysis reveals a pleural fluid/serum protein ratio > 0.9 and a pleural fluid/serum LDH ratio > 0.8. Based on these findings, what is the most likely diagnosis? Cirrhosis with ascites Congestive heart failure Hypoalbuminemia Malignancy Nephrotic syndrome

Correct Answer ( D ) Explanation: Pleural effusions result from the accumulation of fluid between the visceral and parietal pleura. They form as a result of two different processes. Transudative effusions contain little to no protein and result from an increase in hydrostatic pressure or a decrease in oncotic pressure of the pulmonary microvasculature. In contrast, exudative effusions are protein rich. They occur due to increased permeability of the pleura or impaired lymphatic drainage. Light's criteria are used to distinguish transudative from exudative effusions. Out of the answer choices above, malignancy is the only one associated with an exudative effusion. Congestive heart failure (B) is the most common cause of transudative effusions and results from increased hydrostatic pressure. Cirrhosis of the liver (A) and nephrotic syndrome (E) result in transudates due to combined increases in hydrostatic pressure and decreases in oncotic pressure. Hypoalbuminemia (C) can also lead to a transudative effusion due to a significant decrease in oncotic pressure.

A 17-year-old woman presents with a fever, myalgias, and headache. She noted a rash that began 4 days after she noted her fever. The macular rash began on her wrists and ankles and spread toward her chest. She recently returned from hiking the Appalachian Trail in North Carolina about 1 week ago and is concerned about her symptoms. Which of the following is the most likely diagnosis? Babesiosis Colorado tick fever Lyme disease Rocky Mountain spotted fever

Correct Answer ( D ) Explanation: Rocky Mountain spotted fever (RMSF) is a life-threatening infection caused by Rickettsia rickettsia transmitted by dog ticks. Its name derives from its original description in Montana and Idaho in the late 19th century and from the typical petechial rash occurring initially on the wrists and ankles and spreading centripetally (towards the center). It may also involve the palms and soles. Despite its name, most cases are reported from the southeastern and southcentral United States. A little more than 50% of the cases present with the classic triad of rash, fever, and tick exposure, although the rash is rarely present during the first 3 days of the illness and usually appears on Day 4. Abrupt onset of fever, severe headache, and myalgias are the most common presenting symptoms 5-7 days after the tick bite. Babesiosis (A) is a malaria-like illness caused by Babesia microti protozoan. It is transmitted by Ixodes ticks in the northeastern United States; it also has been acquired by transfusion. Like malaria, the protozoan infects red blood cells, causing fever, drenching sweats, myalgias, and headache. It is occasionally associated with a rash. Colorado tick fever (B) is caused by the Coltivirus and transmitted by the dog tick (similar to RMSF) in the western United States. Patients present 3-6 days after a bite with sudden fever, headache, myalgias, and photophobia. A transient petechial rash may occur. In 50% of cases, symptoms resolve and then recur after 3 days. Lyme disease (C) is transmitted by the bite of an Ixodes scapularis tick and is the most common vectorborne disease in the United States. Early infection is associated with the classic erythema migrans rash, a circular lesion with central clearing or erythema at the site of a tick bite.

A 2-year-old boy fails to meet motor milestones. His height is more than 3 standard deviations below the norm on his age-matched growth chart. A brain CT reveals a pituitary gland mass. This patient may be a candidate for which of the following medications? Bromocriptine Desmopressin Pegvisomant Recombinant human growth hormone

Correct Answer ( D ) Explanation: Short stature, often called dwarfism, has a myriad of causes. A more correct medical term for stature three or more standard deviations below the age-appropriate norm is skeletal dysplasia. Achondroplasia and growth hormone deficiency are two of the more common etiologies of short stature, although malnutrition is the most common cause worldwide. There are several causes of growth hormone deficiency, with CNS tumor, surgery or radiation being the most common causes in the pediatric population. Stunted growth, failing to meet height norms or a delay in motor milestones are common presenting symptoms of growth hormone deficiency. Although somewhat controversial in the adults, growth hormone replacement, via recombinant preparations, is the standard of care in the pediatric population, and is most commonly given as a single daily subcutaneous injection. Monitoring of height and bone age is continued every 3-6 months until mature height and normal bone density is obtained. Bromocripitine (A), a dopamine agonist, is used in the treatment of hyperprolactinemia. Although this pituitary condition can be caused by a tumor, it is characterized by amenorrhea, galactorrhea and infertility, not stature abnormalities. Desmopressin (B) is used to treat central diabetes insipidus, not short stature. Pegvisomant (C) is a growth hormone receptor antagonist used in the treatment of acromegaly, a condition of excess, not deficient, growth hormone.

A 45-year-old woman presents for evaluation of right upper quadrant pain. The pain was initially intermittent but has become more constant without radiation. She does not have fever. Her examination is notable for tenderness in the right upper quadrant without a Murphy's sign. She undergoes a right upper quadrant ultrasound as seen above. Which of the following is the most likely diagnosis? Cholangitis Cholecystitis Choledocholithiasis Cholelithiasis

Correct Answer ( D ) Explanation: The ultrasound images display gallstones within the gallbladder consistent with cholelithiasis. This patient has biliary colic, and although the name includes colic, the pain may be a steady constant pain. The physical examination reveals tenderness in the right upper quadrant classically without rebound, guarding or a Murphy's sign. Cholangitis (A) is a clinical diagnosis made by the triad of right upper quadrant pain, fever and jaundice. A gallstone typically causes obstruction leading to bacterial overgrowth. Patients can become ill quickly requiring broad spectrum antibiotics and relief of the obstruction. The ultrasound should demonstrate intra and extrahepatic biliary dilation. Cholecystitis (B) is classically associated with a positive Murphy's sign on physical examination. On ultrasound images, typical findings include a thickened gallbladder wall and pericholecystic fluid. Choledocholithiasis (C) is the presence of a gallstone in the common bile duct. A proximal stone may be visualized on ultrasound although stones in the distal duct may be challenging to identify. Ultrasound may only reveal dilation of the common bile duct as a secondary indication of the stone.

A 16-year-old man is brought to the ED via ambulance after he was found sleeping outside in a snowstorm. The ambient temperature is 25°F. His vital signs are T 36.0°C, BP 125/70 mm Hg, RR 14, and HR 75. He complains of stinging and burning pain in all ten fingers. On exam, you note edema, erythema, and multiple blisters beginning to form over the fingers and hands. Which of the following is the most appropriate next step in management? Administer intravenous antibiotics Debridement of blisters Dry rewarming at 98.6°F (37°C) Wet rewarming at 98.6°F (37°C)

Correct Answer ( D ) Explanation: This patient has evidence of moderately severe (second-degree) frostbite. Rapid rewarming is the cornerstone of frostbite therapy and should be initiated as soon as possible. The injured extremity should be placed in gently circulating water (ideally a whirlpool) at a temperature of 98.6°F to 102.2°F (37°C to 39°C) for approximately 15 to 30 minutes until the distal extremity is pliable and erythematous. The role of prophylactic antibiotics (A) is unclear. Some protocols recommend intravenous penicillin; others recommend topical bacitracin. Nonetheless, the most important initial step is to begin rapid rewarming. The blisters (B) in second-degree frostbite are rich in destructive thromboxane and prostaglandins. Although removal of these blisters results in removal of these destructive substances, not all experts agree that debridement is necessary. Hemorrhagic blisters should never be debrided because this can result in tissue desiccation. However, treatment of the blisters is secondary; the priority is rapid rewarming. Dry rewarming (C) is not recommended and is associated with induction of further injury.

An elderly patient is diagnosed with wet age-related macular degeneration. Which of the following medications is used to treat this disease? Adalimumab Etanercept Infliximab Ranibizumab

Correct Answer ( D ) Explanation: Wet age-related macular degeneration causes a rapidly progressive central visual impairment. It is mainly due to neovascularization and blood accumulation between the choroid and retina. This proliferation of blood vessels is driven by vascular endothelial growth factor (VEGF). As such, antiangiogenic medications, anti-VEGF drugs, are part of a treatment plan for this condition. Other options include laser surgery and photodynamic therapy. These agents are typically injected into the vitreous humour monthly or bimonthly. The initial agent considered was bevacizumab, which is now more commonly used in cancer treatment. A smaller fragment of bevacizumab, ranibizumab (Lucentis®), is now currently used for intraocular injection therapy. Adalimumab (Humira®) (A), etanercept (Enbrel®) (B), and infliximab (Remicade®) (C) are all biologic medications used in treating rheumatoid arthritis, not macular degeneration.

Question: What post-infectious consequence of streptococcal pharyngitis may be prevented with antibiotic therapy?

Answer: Acute rheumatic fever.

Question: True or false: Bacterial infections are the most common cause of acute bronchitis?

Answer: False.

Question: What is the most common nutritional deficiency in children?

Answer: Iron deficiency.

Question: What class of medication is oseltamivir?

Answer: Neuroaminidase inhibitor. Rapid Review Laryngotracheitis (Croup): Patient will be a non-toxic appearing child, 6 months to 3 years old Complaining of URI symptoms with barky, seal-like cough, inspiratory stridor, low-grade fever Labs will show steeple sign on PA view Most commonly caused by Parainfluenza virus Treatment is steroids, aerosolized epinephrine

Question: What are the manifestations of tertiary syphilis?

Answer: Neurosyphilis, cardiovascular involvement, and gumma formation. Rapid Review Secondary Syphilis Patient will be sexually active With a history of painless chancre five to eight weeks ago Complaining of a rash on palms and soles PE will show lymphadenopathy, brownish-red macules and papules, condyloma lata (flat, greyish plaques) Diagnosis is made by VDRL and RPR Most commonly caused by Treponema pallidum Treatment is IM benzathine penicillin G x 1 dose

Question: In what circumstance is dry heat recommended in the treatment of frostbite?

Answer: None. Dry heat may cause further injury. Rapid Review Frostbite Do not rewarm if possibility of refreezing Rx: address hypothermia first, rapid rewarming of affected part in 37-39°C water

Question: What is the most common organism associated with acute mastoiditis?

Answer: Streptococcus pneumoniae. Rapid Review Mastoiditis Untreated OM Otalgia, fever Mastoid erythema, tenderness Osteitis CN VII damage CT scan temporal bone IV ABX

An 8-year-old girl is being evaluated for a persistent cough. She also has malaise, weight loss, and fatigue. She says her cough is worse at night and interferes with her sleep. She had an unknown illness about 10-days ago that consisted of coryza, lacrimation, and a low-grade fever. However, the cough has persisted and is becoming more severe as she has several repetitive spasmodic bursts of 5-10 coughs occurring about 20 times a day. Which of the following is the most likely cause of this patient's condition? Bordetella pertussis Haemophilus influenza Influenza virus Parainfluenza virus

Correct Answer ( A ) Explanation: This patient most likely has pertussis (ie, "whooping cough"). Pertussis is an acute infection of the respiratory tract caused by Bordetella pertussis. The name pertussis means "violent cough," which describes the most consistent and prominent feature of the illness. The inspiratory sound made at the end of an episode of paroxysmal coughing gives rise to the common name for the illness, "whooping cough." Classic pertussis is most often seen in preschool and school-age children. After an incubation period averaging 7-10 days, an illness develops that is indistinguishable from the common cold and is characterized by coryza, lacrimation, mild cough, low-grade fever, and malaise. After 1-2 weeks, this catarrhal phase evolves into the paroxysmal phase: the cough becomes more frequent and spasmodic with repetitive bursts of 5-10 coughs, often within a single expiration. The frequency of paroxysmal episodes varies widely, from several per hour to 5-10 per day. Episodes are often worse at night and interfere with sleep. Haemophilus influenza (B) can result in bacteremia, meningitis, and epiglottitis, but does not cause pertussis. The influenza virus (C) is the pathogen that commonly results in the flu. Symptoms typically include a high fever, rhinitis, sore throat, myalgia, coughing, and fatigue. Parainfluenza virus (D) typically causes lower respiratory infections (eg, bronchiolitis, interstitial pneumonitis, pneumonia) and is a leading cause of morbidity and mortality in infants during the first year of life.

A 35-year-old man with a history of ulcerative colitis presents to the emergency department with complaints of fever, mild abdominal pain, vomiting and diarrhea. His temperature is 102°F, pulse is 125/min, and blood pressure is 88/52 mm Hg. Abdominal X-ray reveals colonic dilatation. Which of the following is the most appropriate initial step in management? Bowel rest and nasogastric tube Initiation of total parenteral nutrition Obtain CT scan of abdomen and pelvis with intravenous contrast Pain control with intravenous morphine

Correct Answer ( A ) Explanation: Toxic megacolon is a complication of infectious colitis or inflammatory bowel disease and is potentially lethal. Diagnosis is made with evidence on plain radiography of colonic dilatation along with clinical signs such as fever, leukocystosis, anemia, tachycardia, dehydration, hypotension, altered mental status or electrolyte abnormality. Initial management of patients determined to have toxic megacolon should be aggressive due to potential mortality. Patients should be monitored in the intensive care unit with serial abdominal radiography and labs every 12 hours. Bowel rest and nasogastric tube placement are essential first steps in order to assist with gastrointestinal decompression. Initial treatment is medical, but a surgical consult should also be ordered at admission as urgent or emergent presentation may require subtotal colectomy with end-ileostomy, the surgical procedure of choice. Total parenteral nutrition (B) may be considered if the patient is malnourished, although has been shown to be of limited value and is not an initial step in the management of patients with toxic megacolon. Obtain CT scan of abdomen and pelvis with intravenous contrast (C) is not the most appropriate next step. An X-ray already identifies colonic dilation and the clinical scenario is consistent with toxic megacolon. Medications reducing colonic motility such as morphine (D) must be stopped and not administered to patients as part of initial management.

A mother is concerned with her newborn baby boy. She believes he doesn't move his right hip like he moves his left hip. You begin an examination with flexing both hips and knees. You come down to the level of the examination table and look at the heights of the patellae. You notice the right patellae sits much lower than the left. Since you are concerned with developmental dysplasia of the hip, you refer the mother to an orthopedist. In your consultation request, you state which of the following physical examination tests is positive? Barlow Galeazzi Ortolani Yergason

Correct Answer ( B ) Explanation: Developmental dysplasia of the hip (DDH) refers to hip instability caused by inadequate pressure of the femoral head against the acetabulum that leads to a shallow socket. This is associated with an increased risk of future hip dislocation and gait abnormalities. Risk factors include a positive family history, breech birth, and female gender. There are several examination tests used to screen for this condition. The Galeazzi (or Allis) test involves flexing the infant's hips and knees, then looking level at the knee heights. If unequal, DDH and congenital short femur may be present in the lower of the two knees. Early diagnosis of hip instability, and treatment with closed reduction, harnessing (Pavlik) or casting, usually results in a favorable prognosis. Later diagnosis may require surgery to correct gait abnormalities. The Barlow and Ortolani tests are used to test for infantile hip instability. The Barlow test (A) begins with hip flexion and adduction. Lateral force is then applied. The test is positive if this lateral force causes hip dislocation. The Ortolani (C) test begins with hip abduction. A dislocated hip will have less abduction than a properly-seated hip. As further abduction is attempted, a posteriorly-placed examiner's finger pushes anteriorly to lift the head back into the acetabulum, leading to a palpable "clunk" and improved passive abduction, which results in a positive test result. Yergason's test (D) is used in evaluating adolescent or adult.

Which of the following best describes the most common symptoms of acute human immunodeficiency virus infection? Diarrhea and abdominal pain Fever, malaise, myalgias, sore throat, and rash Fever, productive cough, and shortness of breath Most individuals are asymptomatic

Correct Answer ( B ) Explanation: In the United States, an estimated 1.2 million individuals are HIV positive, with approximately 50,000 people newly diagnosed each year. The clinical presentation of acute HIV infection, also known as acute retroviral syndrome, is variable both in type and severity of symptoms. The classic presentation is abrupt onset of a viral-like illness, which develops 10 to 14 days following exposure to the virus. Symptoms may include fever, malaise, sore throat, headache, arthralgias, anorexia, nausea, vomiting, and rash. Physical examination findings vary and may include generalized lymphadenopathy, nonexudative pharyngitis, mild hepatosplenomegaly, mucocutaneous ulcers, and oral thrush. A maculopapular rash on the thorax, face, and limbs may also be present. Due to the nonspecific and self-limited nature of symptoms, many affected individuals do not seek care and thus the true incidence of acute HIV is unknown. However, experts estimate that 60 to 90 percent of individuals who contract HIV develop an acute illness. Identification of acute HIV has tremendous public health implications, as acute infection represents the peak infectivity of the disease, characterized by high levels of viral shedding. Routine HIV tests used in most emergency departments measure anti-HIV antibodies, which are often falsely negative during acute infection. Therefore, when acute HIV is suspected, nucleic acid amplification tests, which directly measure viral RNA in the patient's blood, should be used. Diarrhea and abdominal pain (A) are less common than other symptoms in acute HIV. Fever, productive cough, and shortness of breath (C) are common symptoms of pneumonia. That most individuals are asymptomatic (D) is incorrect because experts believe the majority of patients who contract HIV develop an acute illness.

A 16-year-old boy presents with three days of fever, myalgias, and rash. His rash consists of brownish-red macules and papules scattered over his distal extremities, including his palms and soles. Cervical, axillary, and inguinal lymphadenopathy are also noted on exam. Which of the following is true? Doxycycline is the treatment of choice The rash is a result of a sexually transmitted infection The rash is not contagious The symptoms require treatment for resolution

Correct Answer ( B ) Explanation: The adolescent presents with symptoms and signs of secondary syphilis. Syphilis is caused by Treponema pallidum and is a sexually transmitted infection. Primary infection begins with the appearance of a painless chancre, typically on the genital region. Because it is painless, the chancre may not come to the individual's attention or to medical attention. The chancre self-resolves within a few weeks. Weeks to months later, about 25 percent of infected individuals will develop secondary syphilis. Secondary syphilis produces a wide variety of symptoms, including a rash, fever, headache, malaise, anorexia, and lymphadenopathy. The rash is the most characteristic feature and can display a wide variety of appearances, including papules, macules, plaques, and pustules. The only type of lesion that is uncommon is vesicles. The lesions are classically brown, red, or copper-colored. Occasionally, mucous membranes and the perineum may display condyloma lata, which are flat, greyish plaques. The scalp may also be involved and display patchy, "moth-eaten" alopecia. The liver, kidneys, eyes, and musculoskeletal system may also show signs of involvement. Patients who do not develop secondary syphilis still have latent syphilis, which refers to detectable T.pallidum serologies without signs or symptoms of active infection. Doxycycline is not the treatment of choice (A) for secondary syphilis but is the treatment of choice for Rocky Mountain Spotted Fever (RMSF), which also characteristically involves the palms and soles. However, a patient with RMSF would be expected to be much sicker by the time of rash development on day three to four of the illness. The treatment of choice for all forms of early syphilis, including primary, latent, and secondary syphilis, is intramuscular benzathine penicillin G, which remains in detectable concentrations in the body for up to 30 days after administration. Tetracyclines may be used in penicillin-allergic patients but are not the treatment of choice if penicillin can be tolerated. Notably, the only acceptable treatment of pregnant penicillin-allergic women is penicillin, and so they must undergo desensitization prior to treatment. The rash is highly contagious (C) through direct contact, so personal protective equipment should be used during examination. Notably, the symptoms do not require treatment for resolution (D) and will self-resolve even in the absence of therapy. However, the individual remains infected with Treponema pallidum during this time of latent syphilis unless they have been treated with antibiotics.

A 12-year-old boy presents with high fever, muscle and joint aches, and headache for 2 days. He states he just got back from a camping trip in North Carolina. His exam is unremarkable. Labs are normal except for platelets of 95,000 and serum sodium of 128. Which of the following is the most appropriate next step in management? Obtain a Babesia microti DNA PCR and administer atovaquone and azithromycin Obtain a Rickettsia rickettsii immunofluorescence assay and administer doxycycline Send Lyme disease titers and administer amoxicillin Supportive care

Correct Answer ( B ) Explanation: This patient is suffering from early Rocky Mountain Spotted Fever (RMSF) from Rickettsia rickettsii. RMSF is a tick-borne illness which is endemic to North, South and Central America. Although RMSF cases have been reported throughout most of the contiguous United States, 5 states (North Carolina, Oklahoma, Arkansas, Tennessee, and Missouri) account for over 60% of all cases. Clinically, RMSF is characterized by high fevers, arthralgias, myalgias and a petechial rash, which begins on the ankles and wrists and spreads centrally. Early in the disease, there is often no rash and some patients with confirmed RMSF never develop a rash. Thus, early RMSF presents similar to a non-specific viral illness. The clinician may be steered towards the diagnosis of a RMSF by the presence of thrombocytopenia and hyponatremia. R. rickettsii is an obligate intracellular bacteria transmitted to humans by the American dog tick (Dermacentor variabilis) and the Rocky Mountain wood tick (Dermacentor andersoni). RMSF can progress to involve the cardiopulmonary system (myocarditis, AV blocks, dysrhythmias, interstitial pneumonitis, pulmonary edema etc.), nervous system (encephalomyelitis, meningitis, cerebral thrombovasculitis etc.) and cutaneous (vasculitis, ecchymosis and ulcerations). Ricketsial antibodies can be performed but indirect immunofluorescence assay (IFA) is the standard test for diagnosis as it has a high sensitivity (95%). Treatment is most effective when started early in the course of disease. Doxycycline 100 mg twice a day for 7-10 days is the most common therapy but chloramphenicol can be substituted in those with a severe reaction to tetracycline antibiotics. Lyme disease (C) is endemic in many mid-Atlantic states and the Midwest, but is uncommon in North Carolina. The first stage of Lyme disease is classically associated with erythema migrans (bulls-eye rash). Supportive care (D) would be indicated for a viral syndrome which RMSF mimics early in disease. Babesiosis (A) should always be on the differential diagnosis with RMSF and can cause thrombocytopenia but does not cause hyponatremia.

Question: What is the recommended blood pressure goal in patients with hypertension?

Answer: 139/89 mm Hg or less. Rapid Review Hypertension: Eighth Joint National Committee (JNC 8) Recommendations PreHTN: systolic blood pressure (SBP) 120-139 mmHg or diastolic blood pressure (DBP) 80-89 mmHg Stage I HTN: SBP 140-159 mmHg or DBP 90-99 mmHg Stage II HTN: SBP >160 mmHg or DBP >100 mmHg Treatment goals: >60 years: SBP <150, DBP <90 All others: SBP <140, DBP <90 1st line rx for general population: thiazide, CCB, ACEI, or ARB 1st line rx for African Americans: CCB or thiazide Chronic kidney disease: Rx should include ACEI or ARB

Question: How long should an animal be confined in isolation for the evaluation of rabies?

Answer: 5 days. Rapid Review Rabies Patient with a history of exposure to raccoons, bats, or skunks Complaining of hydrophobia, agitation, spasms Treatment is wound care (scrubbing), Ig at wound site, vaccination

Question: What percentage of patients with a tick-borne illness recall a tick bite?

Answer: 50-70%. Rapid Review Rocky Mountain spotted fever (RMSF) Patient with a history of recently in the woods hiking or camping Complaining of abrupt onset of severe headache, photophobia, vomiting, diarrhea, and myalgia PE will show maculopapular eruption on the palms and soles Diagnosis is made by skin biopsy Most commonly caused by Rickettsia rickettsia Treatment is ALWAYS doxycycline, even in children

Question: Electrocautery during surgery can inhibit pacemaker function. How can the pacemaker be changed into an asynchronous mode during the procedure?

Answer: A magnet can safely be placed over the pacemaker during surgery.

Question: How long after initiation of therapy can ACEI-induced angioedema occur?

Answer: ACEI-induced angioedema most frequently occurs in the first month of therapy but can occur up to 10 years after medication initiation. Rapid Review Angioedema Edema: GI and respiratory tracts Tongue, face, neck Idiopathic cause: ACE inhibitors (most common) Hereditary cause: C1 esterase inhibitor deficiency Hereditary rx: C1 esterase inhibitor replacement or FFP Airway management

Question: What is a complication of growth hormone replacement in the pediatric population?

Answer: Benign intracranial hypertension (idiopathic intracerebral hypertension), marked by headache and vomiting within the first 4 months of replacement therapy. Rapid Review Growth Hormone Deficiency Most common cause: pituitary tumor ↓ Muscle mass ↓ Bone density ↑ Lipids ↓ Memory ↓ IGF-1

Question: What type of cell does HIV infect?

Answer: CD4+ T lymphocytes. Rapid Review HIV Infection: Patient will be complaining of abrupt onset of a viral-like illness, fever, malaise, sore throat, headache, arthralgias, anorexia, nausea, vomiting, and rash Diagnosis is made by nucleic acid amplification tests Treatment is HAART

Question: Which disease is transmitted by the reduviid bug?

Answer: Chagas disease. Rapid Review Dengue Fever Patient with a history of recent travel to tropical regions Complaining of "breakbone fever" - high fever with a biphasic pattern, myalgias, and backache Labs will show thrombocytopenia, elevated LFTs and leukopenia Most commonly caused by Aedes aegypti mosquito Treatment is supportive

Question: Which type of gallstone may not be visible on plain film or CT imaging?

Answer: Cholesterol stones do not contain calcium and therefore may be radiolucent. Rapid Review Cholelithiasis Patient will be an obese woman 40 - 50 years old Complaining of slowly resolving right upper quadrant pain that begins suddenly after eating a fatty or large meal Diagnosis is made by ultrasound Most commonly made of cholesterol Treatment is observation or cholecystectomy Comments: Four "F's": Female, Forty, Fat, Fertile

Question: What other antifungal agent may be used in the treatment of oropharyngeal candidiasis?

Answer: Clotrimazole.

Question: Developmental dysplasia of the hip occurs with increased incidence if an infant also has which neck condition?

Answer: Congenital torticollis ("wry-neck", most commonly caused by sternocleidomastoid fibrosis). Rapid Review Developmental Dysplasia of the Hip (DDH) ↓ Pressure of the femoral head against the acetabulum → shallow socket Galeazzi test: flexing the infant's hips and knees to compare knee heights Dx: < 4 mos: ultrasound Rx: Pavlik harness

Question: Which neurosurgical procedure is used for advanced, refractory Parkinson's disease?

Answer: Deep brain stimulation of the subthalamic nucleus. Rapid Review Parkinson's Disease Lewy bodies, substantia nigra dopaminergic neuron loss TRAP: Tremor (resting, "pill rolling"), Rigidity, Akinesia, Postural instability Carbidopa/levodopa, anticholinergic drugs Avoid antipsychotics

Question: Which two common medical conditions need to be evaluated in a patient with a new diagnosis of depression?

Answer: Diabetes and lipid disorders. Rapid Review Bipolar disorder Lifelong, extreme mood swings Mania, hypomania Major depressive disorder Inflated self esteem Decreased need for sleep Pressured speech Flight of ideas Excessive pleasurable activity

Question: What is the common pathogen to cause epididymitis in men older than 35 years of age?

Answer: Escherichia coli. Rapid Review Epididymitis Patient will be complaining of gradual onset unilateral scrotal pain PE will show increased color flow on doppler, relief with testicular elevation (Prehn's sign) Most commonly caused by < 35 yo: C. trachomatis, N. gonorrhea > 35 yo: E. coli, Pseudomonas Treatment is < 35 y/o ceftriaxone/doxycycline, > 35 y/o ciprofloxacin

Question: How long should patients diagnosed with chlamydia or gonorrhea be counseled to avoid sexual contact for?

Answer: For 7 days after completion of antibiotic treatment. Rapid Review Syphilis Primay: painless chancre Secondary: lymphadenopathy, condyloma lata, rash on palms/soles Tertiary: gummas VDRL and RPR positive 4-6 weeks after infection Primary/secondary: IM benzathine penicillin G x 1 dose Tertiary: IM benzathine penicillin G qwk x 3 weeks

Question: What type of bacteria should empiric therapy for toxic megacolon cover?

Answer: Gram-negative and anaerobic bacteria. Rapid Review Toxic Megacolon Most common cause: inflammatory bowel disease Colon dilated > 6 cm on radiographs Systemic toxicity High risk of perforation Rx: IVF, ABX, IV corticosteroids, emergent surgical consultation

Question: What can significantly decrease the risk and severity of head injuries among all-terrain vehicle (ATV) riders?

Answer: Helmets.

Question: Which serotype of the parainfluenza virus is the most common cause of croup?

Answer: Human parainfluenza virus type 1 (hPIV-1) is the most common cause of croup.

Question: What is the treatment for seizure caused by hyponatremia?

Answer: Hypertonic (3%) saline.

Question: When performing a thoracentesis, should the needle enter the thorax above or below the rib?

Answer: Insertion should occur above the rib to avoid the neurovascular bundle that lies along the inferior rib margin. Rapid Review Pleural Effusion Transudate: CHF (most common) Exudate: infection > malignancy, PE ↓ Breath sounds + dull percussion + ↓ tactile fremitus CXR: blunting of the costophrenic angle

Question: How does insulin work to treat hyperkalemia?

Answer: Insulin drives potassium intracellularly to transiently drop serum potassium levels. Rapid Review Hyperkalemia Patient with a history of renal failure, DKA, rhabdomyolysis, tumor lysis Complaining of lethargy, weakness, paralysis PE will show bradycardia, hypotension, cardiac dysrhythmia ECG will show peaked T waves, prolonged PR, wide QRS Treatment is calcium gluconate, insulin, albuterol, kayexalate, bicarbonate

Question: What effect does the Valsalva maneuver have on the murmur associated with aortic stenosis?

Answer: It decreases the murmur. Rapid Review Hypertrophic Cardiomyopathy Asymmetric LV septal wall hypertrophy → outflow obstruction Autosomal dominant (familial form) Young patient Exertional syncope Sudden cardiac death S4 gallop Midsystolic murmur (↑ as preload ↓) Rx: ßBs or CCBs

Question: Do patients with relapsing-remitting MS ever covert to other types of MS?

Answer: Most patients presenting with relapsing-remitting MS convert to secondary-progressive MS after 20 to 40 years. Rapid Review Multiple Sclerosis Demyelinating disorder Caucasian females Optic neuritis (afferent pupillary defect, pain with eye movement, monocular vision loss, pale optic disc) Internuclear opthalmoplegia Lhermitte phenomenon: spinal electric shock sensation with neck flexion CSF: ↑ IgG protein, WBC pleocytosis Rx: methylprednisolone

Question: What gram stain scoring system provides a more sensitive and specific diagnosis than wet mount?

Answer: Nugent's score. Rapid Review Bacterial Vaginosis Patient will be complaining of malodorous vaginal discharge PE will show thin, gray/white discharge Labs will show pH > 4.5, clue cells Diagnosis is made by KOH to smear → fishy odor, "whiff test", Amsel Criteria Most commonly caused by Gardnerella vaginalis Treatment is metronidazole

Question: How is Rocky Mountain spotted fever diagnosed?

Answer: PCR can be used initially, and serial serologic examinations by indirect fluorescent antibody confirm the diagnosis, retrospectively. Rapid Review Rocky Mountain spotted fever (RMSF) Patient with a history of recently in the woods hiking or camping Complaining of abrupt onset of severe headache, photophobia, vomiting, diarrhea, and myalgia PE will show maculopapular eruption on the palms and soles Diagnosis is made by skin biopsy Most commonly caused by Rickettsia rickettsia Treatment is ALWAYS doxycycline, even in children

Question: What is the most common cause of croup (ie, laryngotracheobronchitis)?

Answer: Parainfluenza. Rapid Review Laryngotracheitis (Croup) Patient will be a non-toxic appearing child, 6 months to 3 years old Complaining of URI symptoms with barky, seal-like cough, inspiratory stridor, low-grade fever Labs will show steeple sign on PA view Most commonly caused by Parainfluenza virus Treatment is steroids, aerosolized epinephrine

Question: What is the definitive treatment for sick sinus syndrome?

Answer: Permanent pacemaker placement. Rapid Review Sick Sinus Syndrome (SSS) SA node dysfunction Tachycardia-bradycardia syndrome: sinus rate varies from fast to slow and back again Syncope, palpitations Definitive rx: pacemaker placement + rate control medication Untreated SSS → sinus block or sinus arrest

Question: Antiplatelet therapy in acute coronary syndrome is focused on treating which pathophysiologic process?

Answer: Plaque thrombosis. Rapid Review Ischemic Heart Disease #1 cause of death in USA RFs: family hx, smoking, HTN, DM, cholesterol, male, age > 55 Stable angina: activity → chest pain (CP), relieved by rest, NTG Unstable angina: CP at rest CP, dyspnea, diaphoresis, nausea, hiccups, radiation to shoulder/jaw/back Elderly, diabetics, females, hx of stroke or HF: ↑ risk for atypical presentation Earliest ECG sign of MI: hyperacute T waves Up to 50% of ECGs are negative or nonspecific Highest S/S: troponin I

Question: What are two common complications of untreated recurrent patellar instability?

Answer: Quadricep weakness and patellar arthrosis. Rapid Review Patellofemoral Syndrome Female athletes Knee pain worse after prolonged sitting, bending motions Patella compression test: patella compression into femoral groove → pain, crepitus Apprehension test: patella pushed laterally → quadriceps contracts involuntarily ↑ Q angle Rx: NSAIDs, quadriceps-strengthening exercises

Question: What is the most important independent predictor of mortality for patients with spontaneous bacterial peritonitis?

Answer: Renal dysfunction. Rapid Review Spontaneous Bacterial Peritonitis Patient will have a history of chronic liver disease or cirrhosis Complaining of fever, chills, and abdominal pain PE will show ascites, shifting dullness Labs will show PMNs > 250, WBC >1,000, pH <7.34 Diagnosis is made by analysis of the ascitic fluid Most commonly caused by E. coli, Streptococcus spp Treatment is immediate IV antibiotics (third-generation cephalosporin)

Question: What sexually transmitted infection can present with a rash similar to pityriasis rosea?

Answer: Secondary syphilis. Rapid Review Pityriasis Rosea Patient with a history of a larger lesion 1 week prior, ​"Herald Patch" Complaining of rash on the back PE will show diffuse papulosquamous rash on the trunk, "Christmas tree-like" distribution Treatment is self-limiting disease, itching with antihistamines

Question: List some causes of left bundle branch block?

Answer: Senile fibrosis of the conduction system, chronic hypertension, chronic cardiac ischemia, chronic congestive heart disease and valvular disease. Rapid Review Left Bundle Branch Block QRS duration of > 120 ms Dominant S wave in V1 Broad R wave in lateral leads Absence of Q waves in lateral leads

Question: What is the string of pearls sign?

Answer: Small round pockets of air line up (look like a string of pearls) on the abdominal plain film in the setting of a small-bowel obstruction. Rapid Review Small Bowel Obstruction Causes: adhesions (pelvic surgery) > tumor > hernia (inguinal) Proximal: bilious vomiting Distal: feculent vomiting High pitched bowel sounds X-ray: dilated bowel, air fluid levels, "stack of coins" or "string of pearls" sign Modality of choice: CT NGT, surgery

Question: On which side of the valve is the vegetation typically located in infectious endocarditis?

Answer: Superior. Rapid Review Endocarditis Patient will be complaining of fever, rash, cough and myalgias PE will show Fever, Roth spots, Osler nodes, Murmur, Janeway lesions, Anemia, Nailbed hemorrhages, Emboli (FROM JANE) Diagnosis is made by echocardiography and Duke's criteria Most commonly caused by: IVDA: S. aureus, tricuspid Native valve: Streptococci, mitral Treatment is antibiotics Comments: GI malignancy: S. bovis

Question: What radiographic sign is usually seen in patients with epiglottitis?

Answer: The "thumb sign"; typically seen on lateral C-spine X-ray. Rapid Review Epiglottitis Patient will be complaining of rapid onset of fever and dysphagia PE will show patient leaning forward, drooling, inspiratory stridor Imaging will show "thumbprint" sign Most commonly caused by H. influenzae, Streptococcus Treatment is IV antibiotics and airway management

Question: What is a corpus luteum cyst?

Answer: The corpus luteum of the ovary supports the pregnancy by secreting beta-hCG/progesterone during the first 6-7 weeks. It often becomes cystic. Rapid Review Ultrasound: Intrauterine Pregnancy (IUP) Criteria for dx: yolk sac (YS) within a gestational sac (GS), intrauterine fetal pole, or intrauterine fetal heart activity IUP seen on transvaginal ultrasound > 38 days after LMP or ß-hCG > 1500 IUP seen on abdominal ultrasound > 45 days after LMP or ß-hCG > 4000 YS: Present at 5 to 6 weeks with b-hCG > 2000; first definitive sign of IUP Double decidual sign distinguishes between IUP and a pseudogestational sac

Question: Due to increasing resistance, the CDC does not recommend which antibiotics for routine or alternative treatment of urethritis?

Answer: The quinolones (cipro-, levo- and ofloxacin), due to resistance of Neisseria spp. Rapid Review Gonococcal Urethritis Patient will be complaining of purulent uretheral discharge and dysuria Labs will show gram-negative diplococci Diagnosis is made by: Gold standard - culture on Thayer-Martin media; PCR test is both sensitive and specific Most commonly caused by Neisseria gonorrhoeae Treatment is Ceftriaxone 250 mg IM AND Azithromycin 1 g PO or doxycycline 100 mg BID x 7 days

Question: How long does immunity last after vaccination or clinical infection?

Answer: Three to eight years after vaccination and 15 years after clinical infection. Rapid Review Pertussis (Whooping Cough) Patient with a history of nasal congestion, cough, and low-grade fever Complaining of "rapid fire," repetitive coughing followed by an inspiratory "whoop" and post-tussive emesis Most commonly caused by Bordetella pertussis Treatment is macrolide - azithromycin

Question: What is a first line treatment for psoriasis?

Answer: Topical corticosteroids. Rapid Review Psoriasis Silvery scales Extensor surfaces Nail pitting Auspitz sign: scale removal produces blood droplets Arthritis

Question: What cranial nerve disorder is commonly characterized by bouts of sharp stabbing pain on one side of the face?

Answer: Trigeminal neuralgia. Rapid Review Migraine Headache Gradual onset, unilateral > bilateral, throbbing, pulsating First episode: < 30 years old F > M Triggers: cheese, OCPs, pregnancy, menstruation Without aura: most common, N/V, photophobia, phonophobia Aura: scotoma, flashing lights, sounds Abortive rx: triptans, DHE, antiemetics, NSAIDs Ppx: ßBs. CCBs, TCAs Triptans, DHE: contraindicated in HTN or CV disease

Question: What is the most common cause of chronic retropharyngeal abscesses?

Answer: Tuberculosis.

Question: What urinary symptoms typically accompany the vaginal symptoms of atrophic vaginitis?

Answer: Urinary frequency, dysuria and recurrent infections. Rapid Review Atrophic Vaginitis Patient will be a postmenopausal woman Complaining of dyspareunia, dryness, bleeding, itching PE will show pale, dry, shiny epithelium Most commonly caused by decrease in estrogen Treatment is lubricants, moisturizers, topical estrogen (2nd line)

Question: How is phototherapy performed for the treatment of wet acute macular degenration?

Answer: Verteporfin (Visudyne®) is injected intravenously and attaches to the choroidal vessels. An external light is then administered, activating the verteporfin which destroys neovascularization. Rapid Review Macular Degeneration Patient will be older Complaining of bilateral, gradual central field vision loss PE will show Dry macular degeneration (85% of cases): Atrophic changes and yellow retinal deposits (Drusen spots) Wet macular degeneration: Vascular changes Diagnosis is made by Amsler grid Most common cause of blindness in the elderly

A 3-year-old boy presents to the emergency department with sore throat, irritability, pain with swallowing and fever. On exam, he has a temperature of 103°F, a heart rate of 130 and appears irritable but alert. His neck is tender and rigid and he has unilateral swelling in his posterior pharynx. What of the following is the most appropriate next step? CT scan of the neck Lateral neck X-ray MRI with gadolinium enhancement of the neck Ultrasound of the neck

Correct Answer ( A ) Explanation: A CT scan with IV contrast is currently the imaging modality of choice for a retropharyngeal abscess (RPA), which shows up as a hypodense lesion with peripheral ring enhancement. CT scans are more than 90% sensitive for identifying an RPA with a positive predictive value of about 80% and negative predictive value nearing 100%. A RPA is located in the space between the posterior pharyngeal wall and vertebral bodies. This space communicates with the mediastinum and so mediastinitis can be a severe but rare complication of RPA. Symptoms typically include sore throat, fever, neck stiffness, odynophagia, neck swelling and poor oral intake. Signs include cervical adenopathy, fever, neck stiffness, and a palpable neck mass. Labs usually demonstrate leukocytosis. Blood cultures are indicated prior to antibiotics but usually are negative. Management at first focuses on assuring upper airway patency, stabilizing respiratory distress and fluid resuscitating the patient. An ENT specialist should be consulted as soon as possible for drainage of the abscess. The patient should be continued on antibiotics to cover aerobes, anaerobes and gram-negative organisms. A lateral neck X-ray (B) shows widening of the retropharyngeal soft tissue (more than 7 mm at C2 and more than 14 mm at C6) in nearly 90% of patients with a retropharyngeal abscess (RPA), however, the image is best taken during inspiration with the neck held in normal extension, which may be unreliable for the pediatric population. An MRI (C) is usually not clinically indicated because of the amount of time the child with a suspected RPA and possible airway compromise would be in an unmonitored setting. An ultrasound (D) in experienced hands can be very useful in identifying an RPA, but is not yet accepted as first line overall.

A 23-year-old man presents to your office with a 2-day history of dull, achy, right testicular pain. He reports that the pain began gradually, reaching a peak last night. He denies any trauma and any urethral complaints. Your examination reveals an extremely tender right testis with some tenderness radiating to the epididymis. An ultrasound examination shows an enlarged, heterogeneous right testis with increased color flow. Which one of the following is the preferred management? Antibiotic treatment Emergent urology referral Repeat ultrasonography in 24 hours Watchful waiting

Correct Answer ( A ) Explanation: Acute epididymitis is often the result of descending infection caused by urinary tract pathogens. Men with epididymitis and orchitis typically present with a gradual onset of scrotal pain and symptoms of lower urinary tract infection, including fever. When the infection involves the epididymis and testis, sonography will frequently show an enlarged heterogeneous testis with increased color flow. In sexually active men under age 35, acute epididymitis is caused most frequently by Chlamydia trachomatis and less commonly by Neisseria gonorrhoeae. Laboratory studies, including urethral Gram stain, urinalysis and culture, and polymerase chain reaction assay for C. trachomatis and N. gonorrhoeae, help guide therapy. Initial outpatient therapy is empirical and targets the most common pathogens. Appropriate antibiotics for Neisseria gonorrhoeae and Chlamydia trachomatis respectively are, ceftriaxone (250 mg intramuscularly) and doxycylcine (100 mg twice a day for a duration of 14 days). Additional antibiotic coverage may be indicated based on the patient's sexual history. Unilateral absent flow on color and spectral Doppler sonography is a highly sensitive and specific finding in acute testicular torsion and emergent urology referral (B) is indicated. There is no role for repeat ultrasonography (C) or watchful waiting (D) in patients with acute epididymo-orchitis.

An elderly patient is admitted to the hospital for multiple episodes of syncope. Telemetry reveals periods of sinus bradycardia in the 30s followed by a tachydysrhythmia with heart rates that fluctuate in the 100 to 160 range. During several of the bradycardia episodes the patient becomes hypotensive and symptomatic with shortness of breath, lightheadedness, and dizziness. Which of the following is the most likely underlying tachydysrhythmia associated with this patient's most likely diagnosis? Atrial fibrillation Atrial flutter Multifocal atrial tachycardia Sinus tachycardia

Correct Answer ( A ) Explanation: Atrial fibrillation is the most common tachydysrhythmia associated with sick sinus syndrome. Sick sinus syndrome is defined by electrocardiographic criteria because clinical signs and symptoms, if present, are highly variable. The electrocardiographic characteristics of the sick sinus syndrome include frequent periods of inappropriate, and often severe bradycardia, sinus pauses, arrest, and sinoatrial exit block, often without appropriate atrial and junctional escape rhythms. The failure of escape pacemakers may lead to symptoms including syncope. Alternating bradycardia and an atrial tachydysrhythmia is found in over 50 % of cases. Atrial fibrillation is most common, but atrial flutter and paroxysmal supraventricular tachycardias may also occur. The most common cause of sick sinus syndrome is the replacement of sinus node tissue by fibrous tissue. Symptoms of lightheadedness, presyncope, or syncope are often the reason that the patient seeks medical attention. Other manifestations include increasing dyspnea on exertion, worsening angina, and, in patients with alternating bradycardia and tachycardia, palpitations and other symptoms associated with a rapid heart rate. Atrial flutter (B), sinus tachycardia (D) and paroxysmal supraventricular tachycardia are also associated with sick sinus syndrome; however they are not as common as atrial fibrillation. Atrial flutter is a common dysrhythmia that is associated with similar conditions as atrial fibrillation and are sometimes co-existent. Sinus tachycardia is also associated with this sick sinus syndrome. However, the most common causes of sinus tachycardia are the normal response to exercise and conditions such as fright, anger, or stress. Multifocal atrial tachycardia (C) is not generally associated with sick sinus syndrome. It is defined as a heart rate over 100/min involving at least three distinct P-wave morphologies and is most commonly seen in those with significant lung disease.

A 26-year-old man presents with a severe retro-orbital headache, a sudden-onset fever of 103.3°F, nausea and severe myalgias six days after returning from Panama. On exam, he has a morbiliform rash on his abdomen and back. Which of the following is most likely responsible for his symptoms? Dengue fever Japanese encephalitis Malaria Yellow fever

Correct Answer ( A ) Explanation: Dengue fever is the second most important tropical, febrile illness after malaria. Dengue has a short incubation period of 4-7 days. It is transmitted by the Aedes aegypti mosquito. Travelers returning from the Africa, the Americas and the Indian subcontinent tend to have classic dengue. Those returning from Southeast Asia usually have the hemorrhagic variant. Classic dengue manifests as sudden-onset of high fever, with retro-orbital headache, nausea, vomiting, severe myalgias and a rash. It is also known as "breakbone fever" due to the severe myalgias. Hemorrhagic dengue can result in death due to fever and shock. Diagnosis is clinical and confirmed with ELISA for IgM. Treatment is supportive with both variants. Japanese encephalitis (B) presents with a headache, sudden high fever, nuchal rigidity, vomiting and seizure. Pyramidal and extrapyramidal signs may develop after the fever. The incubation period is 5 to 15 days. It is transmitted via the Culex mosquito. It is rare in travelers because it is found in rice paddies. Diagnosis is clinical and confirmed with viral isolates from the CSF. It can be fatal. Malaria (C) is the diagnosis of any fever >38.5°C (101.4°F) in a traveler returning from a tropical area until proven otherwise. The classic triad of malaria is fever, thrombocytopenia and splenomegaly. Fever is periodic. It is transmitted via the female Anopheles mosquito. Symptoms include myalgias, headache, cough, nausea, vomiting and occasional diarrhea. Diagnosis is clinical and confirmed with blood smear or rapid detection tests. Treatment is supportive and targeted towards eradication of the protozoa. Yellow fever (D) has an incubation period of 3-6 days. It is generally seen in the equatorial zone. The Aedes aegypti mosquito transmits it. Symptoms include fever, myalgias, headache, abdominal pain, conjunctival injection and bradycardia. Many patients recover after this initial presentation. Others relapse into the classic presentation of jaundice, black emesis and albuminuria. Severe cases have high fever, hypotension, shock and arrhythmia. Death can occur within 7-10 days. Diagnosis is clinical. Treatment is supportive. Vaccination is required for travel to endemic areas.

A 54-year-old woman, whose last menstrual period was more than 2 years ago, complains of vaginal dryness and irritation. She denies any recent infection or sexual activity. She is afebrile with normal vital signs. Visual inspection of the vaginal canal reveals pale, dry and shiny epithelium without frank discharge or superficial lesions. Which of the following conditions is the most likely diagnosis? Atrophic vaginitis Bacterial vaginosis Candidiasis Vulvar lichen planus

Correct Answer ( A ) Explanation: Menopause can be defined as amenorrhea for greater than 12 months. Atrophic vaginitis, also referred to as urogenital atrophy, is a major cause of vaginal dryness in postmenopausal women, occurring in about 40% of this population. The generalized loss of estrogens, which is the hallmark hormone alteration associated with menopause, causes thinning of the urogenital epithelium. Vaginal dryness ensues, as well as burning, pruritus, discharge, bleeding and possibly dyspareunia (painful sexual intercourse subsequent to decreased vaginal lubrication). Physical examination of the vaginal canal reveals pale, dry and shiny epithelium without frank discharge or superficial lesions. Diagnosis is mainly clinical, however, it is important to consider infection and cancer in evaluating postmenopausal women with these symptoms. It is unlikely that a patient with a normal temperature and absence of vaginal discharge has any of the common vaginal infections, including bacterial vaginosis (B) and candidiasis (C). Vulvar lichen planus (D) commonly occurs in postmenopausal women, but manifests as an erosive dermatitis associated with intense pruritus, burning and vulvar pain.

Which of the following is correct regarding multiple sclerosis? Becomes more prevalent the farther you live from the equator More common in men Retinal tear is a common presenting symptom Worsens during the course of pregnancy

Correct Answer ( A ) Explanation: Multiple sclerosis (MS) is most common in people of Northern European descent. Caucasians develop MS at nearly twice the rate of African Americans in the United States. In many areas of the world, MS is more prevalent in temperate latitudes (approaching 1 in 500 in some locations) and becomes less prevalent toward the equator (1 in 20,000 or rare case reports only in some locations). Multiple sclerosis is a disease characterized by multifocal areas of demyelination in the brain and spinal cord, with associated inflammatory cell infiltrates, reactive gliosis, and axonal degeneration. It typically presents in young adults with episodic neurologic dysfunction. Pathologically, most cases are characterized by multifocal areas of demyelination and gross gliotic scarring in the brain and spinal cord. Classic locations of these lesions, called plaques, are the optic nerves, periventricular white matter, deep white matter, juxtacortical white matter, corpus callosum, cerebellar peduncles, and dorsolateral spinal cord. The presentation of MS is quite variable. The classic presentation involves acute to subacute onset of impaired vision or sensation. Patients may often describe fatigue, depression, bladder urgency, weakness, impaired balance, and impaired coordination. Multiple sclerosis occurs 2- to 2.5-fold more frequently in women than in men (B), a sex predilection that is common in autoimmune diseases. Optic neuritis, not retinal tear (C) is a classic presenting syndrome, typically with visual symptoms in one eye. Many women with MS successfully have multiple children, and the symptoms of MS diminish during pregnancy (D), especially by the third trimester, when the frequency of exacerbations is reduced by approximately two thirds.

A 14-year-old boy is in the clinic for a well-child visit. He is a straight-A student and plays baseball in school. He eats a healthy and balanced diet. He has a lot of friends in school and enjoys hanging out with them. He likes to ride his bicycle on the weekends. Which of the following is the best advice you would give to the boy and his parents regarding bicycle injuries? Appropriate helmets are those with a firm polystyrene liner that fit properly on the head Helmets effectively reduce the risk of head injury by 55 percent Helmets reduce the risk of brain injury by 48 percent Parents should buy a larger helmet to give the child growing room

Correct Answer ( A ) Explanation: The discussion of bicycle safety should be part of each well-child visit. Pediatricians can be effective advocates for the use of bicycle helmets and should incorporate this advice into their anticipatory guidance schedules for parents and children. Approximately 170 children and adolescents die of injuries incurred while riding bicycles each year in the USA and another 300,000 are treated in emergency departments. Bicycle-related injury is one of the most common reason that children with trauma visit emergency departments. The majority of severe and fatal bicycle injuries involve head trauma. A logical step in the prevention of these head injuries is the use of helmets. Helmets are very effective in reducing the risk of head injury by 85 percent and the risk of brain injury by 88 percent. Helmets also reduce injuries to the mid and upper face by as much as 65 percent. Appropriate helmets are those with a firm polystyrene liner that fit properly on the child's head. Parents should avoid buying a larger helmet to give the child growing room. Helmets effectively reduce the risk of head injury by 55 percent (B) is wrong because helmets reduce the risk by 85 percent. Helmets reduce the risk of brain injury by 48 percent (C) is false because helmet use effectively reduces brain injury by 88 percent. Parents should buy a larger helmet to give the child growing room (D) is wrong because parents should buy a properly fitted helmet.

Which of the following conditions, other than postoperative adhesions, is most likely responsible for causing the diagnosis seen in the image above? Adenocarcinoma Incarcerated hernia Intussusception Volvulus

Correct Answer ( A ) Explanation: The image depicts a small bowel obstruction (SBO). Patients with this condition typically present with diffuse abdominal pain, abdominal distention, and occasionally vomiting. Early on, the pain is mild, crampy, and colicky. As the obstruction progresses, the intestinal contents build up proximally, leading to nausea and vomiting. The intestine distal to the obstruction empties of stool and has decreased peristalsis leading to obstipation. At this time, auscultation may reveal high-pitched, hyperactive bowel sounds. Overall, adhesions, hernias, and cancer account for more than 90% of mechanical SBOs. Postoperative adhesions are the most common cause of SBOs, followed by cancer (20%), and then incarcerated hernias (10%). The classic finding on abdominal radiograph is multiple air-fluid levels and distended loops of small bowel. A paucity of stool and gas is noted distal to the obstruction. Patients should receive supportive measures and fluid resuscitation secondary to depletion of intravascular fluid. A surgeon should be consulted immediately. Hernia (B) is the third-leading cause of small bowel obstruction. Intussusception (C) occurs when an intestinal segment invaginates and telescopes into an adjacent segment of bowel. It is uncommon in the adult population and is seen predominantly in young children. Volvulus (D) is a less common cause of small bowel obstruction but is the second most common cause of large-bowel obstruction after cancer.

A 3-year-old girl is brought to the urgent care clinic for a 4-hour history of dysphagia, fever, drooling, and difficulty breathing. She is in acute distress, restless, and irritable. Her immunizations are up to date. Her temperature is 40.0°C (104°F). Auscultation of the lungs reveals inspiratory stridor. Which of the following is the most likely diagnosis? Acute epiglottitis Anaphylaxis Croup Diphtheria

Correct Answer ( A ) Explanation: This patient most likely has acute epiglottitis. Epiglottitis is inflammation of the epiglottis and adjacent supraglottic structures and results from bacteremia or direct invasion of the epithelial layer by the pathogenic organism. Haemophilus influenzae type b (Hib) is the most common infectious cause of epiglottitis in children. Although the incidence of Hib epiglottitis declined after Hib was added to the routine infant immunization schedule in the United States and other developed countries, Hib epiglottitis still occurs, even in immunized children. Additional causes of epiglottitis in children include other H. influenzae (types A, F, and nontypeable), streptococci, and Staphylococcus aureus, including methicillin-resistant strains. Without treatment, epiglottitis can progress to life-threatening airway obstruction. Anaphylaxis (B) typically develops as a result of an allergic reaction. These patients typically present with swelling of the lips and tongue, urticarial rash, dysphagia, and sometimes inspiratory stridor. Epiglottitis is distinguished from croup (C) by the absence of "barking" cough and the presence of anxiety and drooling. Diphtheria (D) typically has a gradual onset of symptoms (compared to epiglottitis). These patients typically present with sore throat, malaise, and a low-grade fever.

You suspect bipolar I disorder or major depression in a 17-year-old student. Which of the following tools is most appropriate in making one of these diagnoses? Beck Depression Inventory for Primary Care Mood Disorder Questionnaire My Mood Monitor Myers-Briggs Type Indicator

Correct Answer ( A ) Explanation: When staff-assisted depression care is available, screening for depression and bipolar disorders is recommended for patients 12 to 18 years of age. Appropriate screening tools include the age-appropriate Patient Health Questionnaire or the primary care version of the Beck Depression Inventory. The Beck Depression Inventory for Primary Care is adapted from the Beck Depression Inventory, a 21-question multiple-choice self-report inventory and one of the most widely used instruments for measuring the severity of depression. The questionnaire is designed for individuals aged 13 and over, and is composed of items relating to symptoms of depression such as hopelessness and irritability, cognitions such as guilt or feelings of being punished, as well as physical symptoms such as fatigue, weight loss, and lack of interest in sex. The Mood Disorder Questionnaire (B) and the My Mood Monitor (C) are used to rule-out, but not rule-in, a bipolar disorder. The Myers-Briggs Type Indicator (D) is a personality assessment tool. It measures patient preferences in how decisions are made.

Which of the following patients should be classified as having unstable angina? A 51-year-old woman who had chest pain three days ago but now is chest pain free and is found to have a positive troponin with Q waves in leads II, III, and aVF, without ST elevations A 55-year-old woman with a history of hypertension but no prior cardiac disease who complains of one episode of chest pressure that began while pushing her grocery cart and lasted 30 minutes A 65-year-old man with a known history of coronary artery disease who gets chest pain and shortness of breath every time he climbs the steps to his bedroom A 71-year-old man who underwent a coronary catheterization one month ago for early morning chest pain that showed minimal coronary artery disease with no fixed lesions presents with recurrent early morning chest pain that is relieved by nitroglycerin

Correct Answer ( B ) Explanation: Acute coronary syndrome is a spectrum of entities that includes asymptomatic coronary artery disease, stable angina, unstable angina, acute myocardial infarction, and sudden cardiac death. Unstable angina is broadly defined as angina new in onset that occurs at rest or with minimal exertion, or a worsening change in a previously diagnosed stable angina. Unstable angina should be considered a possible harbinger of acute myocardial infarction. A myocardial infarction (A) is acute, evolving, or recent and, unlike anginal syndromes, involves myocardial cell death and necrosis. These patients exhibit a rise and gradual fall in cardiac biomarkers associated with at least one of the following: ischemic symptoms; development of pathologic Q waves on the ECG; ECG changes indicative of ischemia; or coronary artery intervention. Stable angina (C) is transient, episodic chest pain or discomfort, predictable and reproducible with familiar symptoms that occur from a characteristic stimulus. Symptoms improve with rest or nitroglycerin within a few minutes. Variant angina (D), also known as Prinzmetal's angina, is due to coronary artery spasm. This condition can truly be diagnosed only after visualization of the coronary arteries reveal no or minimal fixed lesions. Variant angina occurs at rest and is usually relieved by nitroglycerin or exercise. It can be associated with ST elevations on the ECG that cannot be differentiated from a STEMI.

Which of the following is most commonly associated with acute mastoiditis? Orbital cellulitis Osteitis Otorrhea Trismus

Correct Answer ( B ) Explanation: Acute mastoiditis is usually a complication of untreated or inadequately treated acute otitis media. It has also been described as a complication of leukemia, mononucleosis, sarcoma of the temporal bone, and Kawasaki disease. Acute mastoiditis is a natural extension of otitis media because the mastoid air cells are generally inflamed during acute otitis media. The aditus ad antrum is a narrow connection between the middle ear and mastoid air cells. When this is obstructed, there is increased risk for abscess development and bone destruction. Progression results in destruction of the mastoid bone trabeculae resulting in acute mastoid osteitis. Clinically, patients present with otalgia, fever, headache, and erythema. Pain is universally present. Physical exam findings mimic acute otitis media in addition to postauricular or supra-auricular tenderness and edema. Most cases resolve after administration of antibiotics. For persistent cases, surgical intervention is required for drainage. Orbital cellulitis (A) is a complication of ethmoid sinusitis, not mastoiditis. Otorrhea (C) can occur in mastoiditis (secondary to marked otitis media with ruptured tympanic membrane). However, it is uncommon. Otorrhea is more commonly observed with otitis externa. Trismus (D) is a complication of suppurative parotitis.

Which of the following is an indication for permanent pacemaker placement? Asymptomatic Mobitz type I second-degree AV block Asymptomatic Mobitz type II second degree heart block Asymptomatic sinus bradycardia with heart rate of 40/min Asymptomatic three second sinus pauses

Correct Answer ( B ) Explanation: Asymptomatic Mobitz type II second-degree AV block is an indication for pacemaker placement. This block has a high risk of progressing to complete heart block and should be treated with pacemaker placement, regardless of symptoms. In general, the long term treatment for symptomatic sinus bradycardia or heart block without reversible cause is a permanent pacemaker. These devices are usually placed in the left pectoral area with leads inserted through a vein into the heart. Two general factors guide the decision to place a permanent pacemaker: the association of symptoms with a brady-dysrhythmia and the potential for progression of the rhythm disturbance. Progression is largely dependent on the anatomical location of the conduction abnormality. The location of an AV conduction abnormality, within the AV node or below the AV node in the His-Purkinje system is an important determinant of both the probability and progression rate of conduction system disease. Disease below the AV node, in the His-Purkinje system, is generally considered to be less stable. The most common indications for pacemaker implantation are sinus node dysfunction followed by AV block. Sinus bradycardia in which symptoms such as dizziness, lightheadedness, syncope, fatigue, or poor exercise tolerance are present should be treated. Acquired AV block is the second most common indication for permanent pacemaker placement. Complete, or third-degree AV block, advanced second-degree AV block, symptomatic Mobitz I or Mobitz II second-degree AV block are all indications for pacemaker placement. Asymptomatic sinus bradycardia with heart rate of 40/min (C), asymptomatic Mobitz type I second-degree AV block (A) and asymptomatic three second sinus pauses (D) are not indications for a pacemaker in the absence of symptoms.

A 16-year-old woman presents with vaginal discharge for 4 days. She describes the discharge as gray and malodorous. She denies any pain or itching. On exam, you note vaginal discharge, but no cervical or adnexal tenderness. Microscopy reveals clue cells. Which of the following is the most likely diagnosis? Atrophic vaginitis Bacterial vaginosis Candidiasis Trichomoniasis

Correct Answer ( B ) Explanation: Bacterial vaginosis (BV) is an imbalance in the normal vaginal flora caused by a decrease in the concentration of hydrogen peroxide-producing lactobacillus and an overgrowth of several other microorganisms. These microorganisms vary but could include Gardnerella vaginalis, Mobiluncus spp., Peptostreptococcus spp., and several anaerobic gram negative rods (Prevotella spp., Porphyromonas spp., Bacteroides spp.). Patients usually complain of an amine odor and a grayish or milky, homogenous vaginal discharge. Diagnosis is made by using a diagnostic scale called Amsel's criteria. According to this scale, three of the four criteria must be met to diagnose BV. The four criteria are: an amine odor when potassium hydroxide solution is added to vaginal secretions (otherwise known as a positive whiff test), homogenous vaginal discharge, a vaginal pH >4.5, and the presence of clue cells on microscopy. Clue cells are epithelial cells that are coated with bacteria giving them a stippled appearance. Treatment involves metronidazole or clindamycin either orally or intravaginally. Recurrent BV is common. Atrophic vaginitis (A) is a thinning of the vaginal epithelium caused by a decrease in or lack of estrogen production as seen in prepubertal or postmenopausal women. It is characterized by vaginal dryness, itching, and watery vaginal discharge and can lead to dyspareunia in sexually active women. Candidiasis (C) causes a thick, white vaginal discharge, vulvar erythema, and intense vulvar pruritus. Filaments and spores can be seen on KOH preparations for a definitive diagnosis. Trichomoniasis (D) is a sexually transmitted disease characterized by copious frothy, greenish discharge and vaginal or cervical erythema with petechiae (strawberry spots). Motile flagellates can often be seen on wet mount confirming the diagnosis.

Which of the following best describes the finding seen in the ECG above? Atrioventricular block Left bundle branch block Right bundle branch block Sinoatrial block

Correct Answer ( B ) Explanation: Bundle branch blocks are abnormal conduction abnormalities (not rhythm disturbances) in which the ventricles depolarize in sequence, rather than simultaneously, thus producing a wide QRS complex (> 120 msec) and a ST segment with a slope opposite that of the terminal half of the QRS complex. A left bundle branch block is a bifascicular block in which ventricular activation is by way of the right bundle branch. The impulse travels down the right bundle, activating the septum and the free wall of the right ventricle, and then continues on in the same direction to activate the free wall of the left ventricle. Because the dominant forces are traveling in the same direction, there is a tendency toward monophasic QRS complexes. The ECG in a LBBB will show a large wide R wave in lead I and a negative wave (QS or rS) in lead V1. AV block (A) produces PR interval lengthening, not R-R' complexes. The ECG in a RBBB (C) will show a wide S wave in lead I and a RSR' pattern in lead V1. Sinoatrial block (D) is represented electrocardiographically by a missed cycle or beat within otherwise normal appearing P waves and QRS complexes.

A 22-year-old woman is complaining of recurring headaches that have been worsening over the past several months. They are usually on the right side of her head causing a throbbing pain and she has noticed that they tend to occur more frequently around the time of her menses. She is very concerned because recently she has been seeing "spots" and feels a tingling in her left arm around the time of the headaches. She takes 400 mg of ibuprofen and goes to sleep in order to obtain relief. What is the most likely type of headache? Cluster Migraine Tension Thunderclap

Correct Answer ( B ) Explanation: Migraine headache is a common disorder affecting over 10% of the population. It is more common in women than in men and has several characteristics that can aid in its diagnosis. Migraine is most often unilateral, recurring, throbbing, and associated with certain recognizable triggers such as hormonal fluctuations during menses (there are many other possible environmental triggers). Non-steroidal anti-inflammatory medication and sleep will often relieve the symptoms. Although the presence of a visual, auditory, or sensory aura will help in the diagnosis, most patients do not experience an aura related to their migraine. Cluster (A) headaches are a trigeminal autonomic cephalgia and often result in debilitating pain with activation of the trigeminal autonomic system resulting in symptoms such as runny nose and watery eyes. Tension (C) headache is the most common type of headache. It is usually felt bilaterally or toward the back of the head and neck. Sometimes the musculature around the scalp will feel tender to palpation during a tension headache. There would not be photophobia, phonophobia, or nausea as you would commonly see with a migraine. A thunderclap (D) headache is a severe headache of sudden onset that peaks within a few hours. It is often described as the "worst headache of my life" and does not tend to recur. Thunderclap headaches are worrisome for cerebrovascular events such as subarachnoid hemorrhage and aneurysmal bleeding and should therefore undergo immediate evaluation.

Which antiepileptic medication commonly causes hyponatremia at therapeutic doses? Lamotrigine Oxcarbazepine Phenytoin Topiramate

Correct Answer ( B ) Explanation: Oxcarbazepine causes hyponatremia < 125 in 2.5 percent of patients. Most patients develop hyponatremia within the first three months of initiating oxcarbazepine treatment, though some patients can develop hyponatremia more than one year after initiating treatment. The mechanism is not well understood. The risk increases when patients are on other antiepileptic medications and thiazide diuretics. Neurologists frequently monitor serum sodium levels for the first three months after initiating treatment and advocate checking a serum sodium level on any patient who is on oxcarbazepine and presents with headache, somnolence, drowsiness, or increased seizure frequency. Phenytoin (C) has a narrow therapeutic window, which increases the risk of toxicity. Phenytoin toxicity is characterized by nystagmus, ataxia, drowsiness, and coma. Long-term use is associated with gingival hyperplasia. Topiramate (D) causes a hyperchloremic, nonanion gap, metabolic acidosis in up to 25 percent of patients. This is due to inhibition of carbonic anhydrase and increased renal excretion of bicarbonate. It can also cause weight loss, intermenstrual spotting, and decreased efficacy of oral contraceptives. Lamotrigine (A) has an FDA "black box" warning due to increased risk of Stevens-Johnson syndrome.

Which of the following is true regarding pertussis? Childhood vaccination and clinical disease provide lifelong immunity Infectivity is greatest in the catarrhal stage Lobar consolidations are commonly seen on chest radiography The treatment of choice is oral amoxicillin

Correct Answer ( B ) Explanation: Pertussis (whooping cough) is an acute respiratory illness caused by Bordetella pertussis. Prior to the development of a vaccination, pertussis primarily affected children under the age of 10 years. Since the late 1990s, pertussis has become more prominent in adolescent and adult populations. The clinical course is divided into three stages. The first stage, or known as the catarrhal stage, is often indistinguishable from other respiratory infections. It begins after an incubation period of seven to 10 days and lasts for about a week. Symptoms include rhinorrhea, low-grade fever, malaise, and conjunctival injection. Infectivity is the greatest in the catarrhal stage. Stage two, known as the paroxysmal stage, begins as the fever subsides and the cough develops. Patients have paroxysms of a staccato cough followed by sudden forceful inhalation, producing the characteristic whoop. Only one-third of adult patients develop the whoop, and it is also rare in infants. Infants are at risk of apneic episodes during this stage. The paroxysmal stage lasts two to four weeks. The third, or convalescent stage, lasts several weeks to months and is characterized by a residual cough. Chest X-ray findings include peribronchial thickening and atelectasis. Lobar consolidations (C) are not common and would be concerning for superimposed bacterial infection. Oral amoxicillin (D) is not effective in the treatment of Bordetella pertussis. The antibiotic class of choice includes macrolides. Antibiotic treatment does not significantly reduce the severity or duration of illness but does decrease infectivity. Childhood vaccination and clinical disease do not provide lifelong immunity (A).

A 65-year-old man with cirrhosis and ascites presents to the emergency room with diffuse abdominal discomfort, fever, and altered mental status. His home medications include furosemide and spironolactone. On physical exam he is febrile and his abdomen is mildly tender with positive shifting dullness and fluid wave. You perform a diagnostic paracentesis. Which of the following findings best supports your diagnosis? Bloody appearance of ascites Neutrophil count > 250 pH > 7.34 Polymicrobial growth on ascitic culture

Correct Answer ( B ) Explanation: Spontaneous bacterial peritonitis (SBP) is an infection of ascitic fluid that cannot be attributed to any other intrabdominal conditon. It is one of the most common bacterial infections in patents with cirrhosis and should always be at the top of the differential when examining a tender abdomen in the presence of ascites in a febrile patient. A peritoneal fluid absolute neutrophil count greater than 250 cells is the accepted criterion for the diagnosis of spontaneous bacterial peritonitis. Gram-negative bacteria are the most common pathogens causing spontaneous bacterial peritonitis and most are difficult to culture, therefore a positive culture is not needed for the diagnosis. Treatment is directed primarily at early administration of empiric antibiotics for the duration of 7-10 days. Ascitic fluid pH can also be measured, however it is not diagnostic criteria for spontaneous bacterial peritonitis. Ascites with pH < 7.34 (C) suggests a higher likelihood for SBP. Bloody ascites (A) is more suggestive of SBP, but is not diagnostic. Cultures of the ascitic fluid should be obtained in order to identify the causative organism, however cultures are negative in 50% of patients with spontaneous bacterial peritonitis. Furthermore, more than 92% of all cases with SBP are monomicrobial. A positive polymicrobial culture indicates secondary peritonitis (D) such as a bowel perforation.

A 12-month-old girl is in the clinic for a well-child check. She can walk and is eating more table foods. The parents have given her goat's milk for two months. Physical examination is normal. Laboratory screening obtained shows hemoglobin of 10 g/dL, mean corpuscular volume 77 fL and elevated red cell distribution width. Which of the following is the most likely diagnosis? Folate deficiency Iron deficiency anemia Thalassemia Vitamin B12 deficiency

Correct Answer ( B ) Explanation: The girl's complete blood count shows anemia that has low mean corpuscular volume (MCV) and elevated red cell distribution width that is suspicious for iron deficiency anemia (IDA). The World Health Organization (WHO) estimates that anemia affects one quarter of the world's population and is concentrated within preschool-aged children and women. The majority of the anemia is due to iron deficiency. Risk assessment for iron deficiency for all infants is recommended at 4, 15, 18, 24, and 30 months of age, and annually thereafter. The most important screening test is a focused dietary history, and it is more useful than hemoglobin levels. The primary risk factors for iron deficiency are: history of prematurity or low birth weight, use of low iron formula, or feeding of non-formula cow's milk, goat's milk, or soy milk for the milk-based part of the diet before 12 months of age or fewer than two servings per day of iron-rich foods (meats or fortified infant cereal) after six months of age. The American Academy of Pediatrics suggests universal laboratory screening for iron deficiency at approximately one year of age. A presumptive diagnosis of iron deficiency anemia (IDA) is made by the combination of risk assessment and laboratory screening (hemoglobin < 11 g/dL). A low mean corpuscular volume (MCV) and elevated red cell distribution width (RDW) further support the diagnosis. Folate deficiency (A) and vitamin B12 deficiency (D) would both reveal macrocytic anemia with MCV greater than 100 fL. Thalassemia (C) would also present with microcytic anemia with MCV less than 80 fL, and the family history may be positive. Iron stores are absent and serum ferritin levels are low in iron deficiency, while both are normal or elevated in the thalassemias.

An 11-year-old girl presents with a rash for the past 2 weeks. Her mother states that the rash began as a single red spot on the patient's upper back and then spread to the rest of her body. On physical exam, you note pink maculopapular oval patches on the patient's trunk. What treatment is indicated? Acyclovir Diphenhydramine Topical ketoconazole cream Trimethoprim-sulfamethoxazole

Correct Answer ( B ) Explanation: The patient has pityriasis rosea. It most commonly occurs in the spring and fall seasons and is seen in older children and young adults. The rash initially presents with a single red lesion on the trunk, known as a herald patch. In about one to two weeks, the rash progresses to pink maculopapular oval patches covering the trunk in a characteristic "Christmas tree" pattern. Patients with pityriasis rosea require symptomatic treatment, including diphenhydramine to relieve itching. The rash resolves within 3-8 weeks and is not contagious. Due to the unclear etiology of pityriasis rosea, acyclovir (A), ketoconazole (C), and trimethoprim-sulfamethoxazole (D) are not indicated.

A 16-year old boy presents with a superficial bite to his right arm. He states he was camping in the woods and a bat flew into his tent. Which of the following treatments should this patient receive? Human rabies immunoglobulin and 3 doses of inactivated rabies vaccine over 7 days Human rabies immunoglobulin and 4 doses of inactivated rabies vaccine over 14 days Human rabies immunoglobulin only Inactivated rabies vaccine only

Correct Answer ( B ) Explanation: The rabies virus enters the CNS and causes a wide range of symptoms from headaches, anorexia, hallucinations, agitation to seizures, and hydrophobia. Bats are the most common animals to infect humans. Raccoons are the most common animal infected with rabies. The patient's history and presentation suggest an infection with the rabies virus and post-exposure prophylaxis with the inactivated rabies vaccine and the human rabies immunoglobulin should be administered. The human rabies immunoglobulin is a one time dose where as the inactivated rabies vaccine is given 4 times over 14 days. In addition the bite wound should be examined and cleaned and antibiotics may be of benefit. Pre-exposure prophylaxis should be given to those who live in an endemic area to rabies or those who work with animals that are potentially infected with rabies. The inactivated rabies vaccine (D) is used as pre-exposure prophylaxis. Human rabies immunoglobulin (C) is never given alone. Human rabies immunoglobulin and 3 doses of inactivated rabies vaccine over 7 days (A) is incorrect since 4 doses of inactivated rabies vaccine over a 14-day duration is required for post-exposure prophylaxis.

Which of the following is correct regarding the condition seen in the image above? Involvement of the scalp and nails is uncommon It is associated with the Auspitz sign Lesions most commonly occur on flexor surfaces Systemic steroids are the preferred treatment

Correct Answer ( B ) Explanation: The well-demarcated erythematous plaques and papules with silvery white scales are characteristic of psoriasis. Removal of the scale typically reveals pinpoint-bleeding areas referred to as the Auspitz sign. There is a hereditary predilection for the condition and often begins in the 2nd or 3rd decade of life. Lesions tend to be symmetric and most commonly found on the trunk, scalp, nails (A), and extensor surfaces (C). Systemic steroids (D) should be avoided due to the risk of developing rebound or induction of pustular psoriasis.

A 42-year-old man presents to the Emergency Department with fever, chills, cough, and hemoptysis. He has a history of intravenous opioid use. Vital signs include BP 110/65 mm Hg, HR 120 beats per minute, RR 20 breaths per minute, and T 103.4°F. Chest X-ray is shown above. Which of the following is the most likely diagnosis? Diffuse alveolar hemorrhage Infectious endocarditis Miliary tuberculosis Wegener's granulomatosis

Correct Answer ( B ) Explanation: This patient is presenting with signs and symptoms of infectious endocarditis. Risk factors for infectious endocarditis include rheumatic heart disease, congenital or acquired valvular disease, and intravenous drug use. Infectious endocarditis is classified as acute or subacute depending upon the time course and presentation. Acute endocarditis more often affects normal valves in younger patients. Septic emboli (as shown in the above chest X-ray) and significant illness are common on presentation. Subacute endocarditis has a predilection for abnormal valves and more frequently occurs in older patients. These patients are typically less ill on presentation with intermittent fevers and constitutional symptoms. Left-sided endocarditis involves either the aortic or mitral valve. It is more common than right-sided endocarditis. Organisms often implicated in left-sided endocarditis include Streptococcus viridans, Staphylococcus aureus, and those in the Enterococcus family. Complications include systemic infarcts from septic emboli. Right-sided endocarditis involves either the pulmonic or tricuspid valve. It is classically seen in intravenous drug users. Organisms implicated in right-sided endocarditis include Staphylococcus aureus, Streptococcus pneumoniae, and gram negative bacteria. Presenting symptoms often include fever, cough, hemoptysis, chest pain, and dyspnea. Right-sided endocarditis is frequently misdiagnosed initially as pneumonia. Dermatologic and ocular manifestations of endocarditis are important indicators of the diagnosis. Roth spots are retinal hemorrhages with central clearing seen on funduscopic examination. Osler nodes are painful nodules on fingers and toes. Janeway lesions are painless erythematous plaques on the palms and soles. Splinter hemorrhages occur beneath the nails due to septic emboli. Diagnosis is made by having either both major criteria, 1 major and 3 minor criteria, or 5 minor criteria. Major criteria include 2 positive blood cultures with at least 3 sets sent one hour apart of organisms common to infectious endocarditis or abnormal echocardiography with either visible vegetation, new valvular regurgitation, prosthetic valve dehiscence, or myocardial abscess. Echocardiography is the hallmark of imaging for endocarditis and is preferably done via the transesophageal route. Minor criteria include predisposing risk factors or IV drug use, fever, vascular events such as septic emboli and Janeway lesions, immunologic events such as Osler nodes or Roth spots, echocardiographic findings consistent with endocarditis not meeting major criteria, and positive blood cultures not meeting major criteria. Management includes antibiotics for the suspected organism based on the clinical situation. In an IV drug user, coverage should include methicillin-resistant Staphylococcus aureus and Pseudomonas aeruginosa in addition to the typically implicated organisms. The most appropriate antibiotics for this patient would include cefepime and vancomycin. Diffuse alveolar hemorrhage (A) will often present with cough and hemoptysis; however, fever is uncommon. Chest X-ray will typically show bilateral infiltrates, primarily central. The classic finding in diffuse alveolar hemorrhage is return of increasing amounts of blood on bronchoalveolar lavage. Miliary tuberculosis (C) will often present with a diffuse reticulonodular pattern on chest X-ray as well as generalized malaise and fever, but rarely with hemoptysis. Also, intravenous drug use is a much more significant risk factor for infectious endocarditis. Wegener's granulomatosis (D) will often present with chronic low-grade fevers, night sweats, fatigue, and weight loss. Patients will often have chronic sinusitis, but not present until they have severe hemoptysis, vasculitis, or oral lesions. Chest X-ray will often show bilateral central infiltrates; however, the patient will not present with high fever and intravenous drug use is not a risk factor for this disease.

A four-year-old child presents to the emergency room with high fever, barking cough, stridor and moderate retractions at rest. Which of the following is the most appropriate treatment for this condition? Albuterol nebulizer Ceftriaxone Nebulized racemic epinephrine Oral oseltamivir

Correct Answer ( C ) Explanation: A child with moderate croup has a barking cough, stridor, and moderate retractions at rest. Nebulized epinephrine is the treatment of choice as the rapid local vasoconstriction of the subglottic mucosa reduces swelling and provides rapid improvement. Oral or intravenous steroids should be administered as well to prevent the progression or airway edema as well as rebound edema that may occur up to three hours after the administration of racemic epinephrine. Humidified oxygen can also be given to help control airway edema. Evidence is poor, but in severe cases, children may benefit from heliox. This is a mixture of helium and oxygen which is less dense than air. It assists in air flow through a narrowed airway. In patients that require intubation, an uncuffed endoctracheal tube at least 0.5 mm narrower than the standard for age should be used given concern for significant airway swelling. Albuterol nebulizers (A) should be given in patients with concern for reactive airway disease. It is not indicated in patients with concern for upper airway obstruction. Ceftriaxone (B) is a third-generation cephalosporin that is not indicated in the treatment of viral croup. Oseltamivir (D) is used in the treatment of influenza virus.

A 3-year-old boy is brought to the pediatric emergency department by his mother. She tells you he has had a fever and cough for the past 2 days. The boy appears to be agitated and is in respiratory distress. His temperature is 38.6°C (101.5°F) and his respiratory rate is 35/min. Physical examination shows subcostal retractions. Auscultation of the lungs reveals inspiratory stridor with a prolonged inspiratory phase. Chest X-ray shows subglottic narrowing. Which of the following is the most likely diagnosis? Acute epiglottitis Foreign body aspiration Laryngotracheobronchitis Respiratory syncytial virus

Correct Answer ( C ) Explanation: Based on the constellation of findings, this patient most likely has croup (ie, laryngotracheobronchitis). Croup typically presents with the classic triad of barking cough, stridor, and hoarseness. It is most often caused by a viral infection (parainfluenza). Treatment usually consists of oral steroids (eg, dexamethasone) or nebulized epinephrine for severe cases. Although chest X-rays are typically not required, if done, they will usually show subglottic narrowing (ie, "steeple sign") as in this case. Acute epiglottitis (A) is caused by H. influenza type b. It is typically distinguished from croup by the absence of a barking cough. Additionally, these patients will present with difficulty swallowing and drooling. Foreign body aspiration (B) typically presents with a sudden onset of choking and symptoms of upper airway obstruction. Respiratory syncytial virus (D) usually presents with mild symptoms that are often indistinguishable from common colds or other minor illnesses and is most commonly seen in infants under the age of 1.

Which of the following sexually transmitted infections has a high rate of cotransmission with HIV infection? Gardnerella vaginalis Neisseria gonorrhoeae Treponema pallidum Trichomonas vaginalis

Correct Answer ( C ) Explanation: Disorders characterized by genital ulcers (syphilis, herpes, chancroid, lymphogranuloma venereum, granuloma inguinale) have high rates of HIV cotransmission. Treponema pallidum is the causative agent of syphilis and is associated with a painless ulcer during the initial stage of infection (primary syphilis). Ulcers typically appear 3 to 6 weeks after exposure. Multiple sexually transmitted infections often occur together. There is a high rate of coinfection of chlamydia and gonorrhea (B), as well as trichomoniasis (D). However, coinfection with HIV is greatest in ulcer-causing infections. Gardnerella vaginalis (A) is implemented in bacterial vaginosis, which is generally polymicrobial in nature.

A 7-year-old boy presents with two days of fever, sore throat, and bilateral tender anterior cervical lymphadenopathy. Examination reveals 2+ erythematous tonsils covered in white exudate. Which of the following is his most likely diagnosis? EBV-induced mononucleosis Group A Strep lymphadenitis Group A Strep pharyngitis Viral upper respiratory infection

Correct Answer ( C ) Explanation: Group A Strep pharyngitis is common in children aged 5-15 years, especially in the winter. Common features include exudative pharyngitis, fever, bilateral tender anterior cervical lymphadenopathy, and a lack of cough and coryza. Children may also develop a scarlatiniform "sandpaper" rash, palatal petechiae, non-specific abdominal pain, and vomiting. Even in the presence of these classic clinical features, rapid antigen testing (and throat culture, if negative) is recommended. Notably, the Centor Criteria, which are commonly used to predict the probability of Group A Strep pharyngitis in adults, have not been validated in children. EBV-induced mononucleosis (A) may also present with exudative pharyngitis. However, the associated lymphadenopathy classically involves the posterior cervical nodes, or the lymphadenitis may be generalized. Fever is variably present. Mononucleosis is also much more common amongst teenagers. While the patient does have tender anterior cervical nodes, these nodes more likely represent reactive lymphadenopathy than Group A Strep lymphadenitis (B). Infectious lymphadenitis classically involves one lymph node or multiple adjacent nodes and rarely is bilateral or symmetric. Viral upper respiratory infections (D) commonly cause sore throat, reactive lymphadenopathy, and sometimes fever, but cough and coryza are typically prominent features. When exudative pharyngitis is a prominent feature, testing for Group A Strep should be completed prior to assigning a diagnosis of viral upper respiratory infection.

Which of the following should be assessed in the physical exam of a patient with essential hypertension? Costal vertebral angle tenderness Deep tendon reflexes Ocular fundus Pupillary response

Correct Answer ( C ) Explanation: Hypertension is defined as a systolic blood pressure of 140 to 159 mm Hg or a diastolic blood pressure of 90 to 99 mm Hg on two or more occasions. Hypertension is commonly seen in the primary care setting and risk factors include family history of hypertension, obesity, advancing age, African-American race, physical inactivity, high sodium diet, and diabetes. When taking the history of a patient with hypertension, questions should include duration of the diagnosis, previous treatment, and aggravating factors such as alcohol consumption, smoking, use of nonsteroidal anti-inflammatory agents and other prescription medications. The purpose of the physical exam is to assess for end-organ damage and cardiovascular disease as well as potential causes of secondary hypertension. Physical exam should include an accurate reading of blood pressure along with an assessment of general appearance, heart, lungs, neck, abdomen, and extremities. A fundoscopic exam of the eyes should be used to evaluate for hypertensive retinopathy including hemorrhage, papilledema and cotton wool spots. Costal vertebral angle tenderness (A) is assessed to rule out pyelonephritis or renal stones. Deep tendon reflexes (B) are evaluated in patients as part of a complete neurological exam. Neurological abnormalities seen in patients with hypertension include visual disturbance, focal weakness or confusion. Pupillary response (D) is assessed as part of cranial nerve testing in patients with neurological defects.


Kaugnay na mga set ng pag-aaral

Principles of Financial Accounting

View Set

Human Resource Management Chapter 1

View Set

Fundamentals of Nursing III (Chap 26 Safety, Security, Emergency Preparedness Prep U)

View Set

CHAPTER 31 - MULTIPLE GESTATIONS

View Set

Prokaryotic cells and gram positive/negative

View Set

CA RDA Test Bank - Specialty Dentistry

View Set

Seasonal Influenza Vaccination Training (2hr)

View Set

Chapter 1: Insurance Terms and Related Concepts

View Set